Last visit was: 24 Apr 2024, 19:13 It is currently 24 Apr 2024, 19:13

Close
GMAT Club Daily Prep
Thank you for using the timer - this advanced tool can estimate your performance and suggest more practice questions. We have subscribed you to Daily Prep Questions via email.

Customized
for You

we will pick new questions that match your level based on your Timer History

Track
Your Progress

every week, we’ll send you an estimated GMAT score based on your performance

Practice
Pays

we will pick new questions that match your level based on your Timer History
Not interested in getting valuable practice questions and articles delivered to your email? No problem, unsubscribe here.
Close
Request Expert Reply
Confirm Cancel
SORT BY:
Date
Math Expert
Joined: 02 Sep 2009
Posts: 92900
Own Kudos [?]: 618824 [19]
Given Kudos: 81588
Send PM
Most Helpful Reply
Math Expert
Joined: 02 Sep 2009
Posts: 92900
Own Kudos [?]: 618824 [8]
Given Kudos: 81588
Send PM
General Discussion
avatar
Intern
Intern
Joined: 21 May 2015
Posts: 1
Own Kudos [?]: [0]
Given Kudos: 0
Send PM
RC & DI Moderator
Joined: 02 Aug 2009
Status:Math and DI Expert
Posts: 11170
Own Kudos [?]: 31890 [2]
Given Kudos: 290
Send PM
Re: M18-04 [#permalink]
2
Kudos
Expert Reply
priyankasharma123 wrote:
Hi,

Why can't i change my base as in either BD ,AB,BC.
In this case my height will vary.
Please explain how come you have height to be BE and not (BD,AB,BC).

Thanks


hi,
irrespestive of what height depending on the base we take, the area will remain the same..
Since here we have a height and a base and the area of triangle depends on the base as the height in this case will be the same and thus only one variable involved, Why should we complicate the things by taking a bases , which are not the same resulting in different heights and thus two variables..
User avatar
Senior Manager
Senior Manager
Joined: 31 Mar 2016
Posts: 325
Own Kudos [?]: 195 [1]
Given Kudos: 197
Location: India
Concentration: Operations, Finance
GMAT 1: 670 Q48 V34
GPA: 3.8
WE:Operations (Commercial Banking)
Send PM
Re: M18-04 [#permalink]
1
Kudos
I think this is a high-quality question and I agree with explanation.
Intern
Intern
Joined: 07 Mar 2011
Posts: 10
Own Kudos [?]: 52 [2]
Given Kudos: 5
Send PM
Re: M18-04 [#permalink]
2
Bookmarks
I think this is a high-quality question and I agree with explanation. I have another approach.

Triangle Median property
1. The median of the triangle divides the triangle into two smaller triangles which have the same area.
In this case point D is not a median. Applying above property on statement 1 we can say area of triangle ABD > area of triangle DBC.

Statement 2 clearly not sufficient.
Math Expert
Joined: 02 Sep 2009
Posts: 92900
Own Kudos [?]: 618824 [0]
Given Kudos: 81588
Send PM
Re: M18-04 [#permalink]
Expert Reply
I have edited the question and the solution by adding more details to enhance its clarity. I hope it is now easier to understand.
Intern
Intern
Joined: 18 Jul 2019
Posts: 1
Own Kudos [?]: 0 [0]
Given Kudos: 4
Send PM
Re: M18-04 [#permalink]
Dear Bunuel, how can A be the correct answer if S1 says AD<DC ? Could you please clarify ?
Thanks .

Posted from my mobile device
Math Expert
Joined: 02 Sep 2009
Posts: 92900
Own Kudos [?]: 618824 [0]
Given Kudos: 81588
Send PM
Re: M18-04 [#permalink]
Expert Reply
bsolly wrote:
Bunuel wrote:
Official Solution:


If, in triangle \(ABC\), angle \(ABC\) is the largest and point \(D\) lies on segment \(AC\), is the area of triangle \(ABD\) larger than that of triangle \(DBC\)?

Consider the diagram below:



Notice that \(BE\) is the height of triangle \(ABC\). Now, the area of triangle \(ABD\) is \(\frac{1}{2}*height*base = \frac{1}{2}*BE*AD\), and the area of triangle \(DBC\) is \(\frac{1}{2}*height*base=\frac{1}{2}*BE*DC\). So, we can see that the area of triangle \(ABD\) will be greater than the area of triangle \(DBC\) if \(AD\) is greater than \(DC\).

(1) \(AD \lt DC\). Sufficient.

(2) \(AB \lt BC\). Not sufficient.


Answer: A



Dear Bunuel, how can A be the correct answer if S1 says AD<DC ? Could you please clarify ?
Thanks .

Posted from my mobile device


The area of triangle \(ABD\) will be greater than the area of triangle \(DBC\) if \(AD\) is greater than \(DC\). (1) says that \(AD \lt DC\). Hence, the area of triangle \(ABD\) is NOT greater than the area of triangle \(DBC\). We have a definite NO answer to the question. Sufficient.

P.S. Recall that a definitive 'no' is as valid as a 'yes' in establishing sufficiency. The critical factor in GMAT Data Sufficiency is not whether the answer is positive or negative, but whether the data provided conclusively supports that answer. A consistent 'no' can effectively fulfill the question's requirement, confirming the sufficiency of the data.
Manager
Manager
Joined: 15 Feb 2021
Posts: 187
Own Kudos [?]: 220 [0]
Given Kudos: 13
Send PM
Re: M18-04 [#permalink]
Bunuel, what is the importance of angle ABC being the largest one? Would the answer change if that angle weren't the largest?
RC & DI Moderator
Joined: 02 Aug 2009
Status:Math and DI Expert
Posts: 11170
Own Kudos [?]: 31890 [0]
Given Kudos: 290
Send PM
Re: M18-04 [#permalink]
Expert Reply
andreagonzalez2k wrote:
Bunuel, what is the importance of angle ABC being the largest one? Would the answer change if that angle weren't the largest?



The answer will not change. ABC being the largest angle does not have any affect on the question.
GMAT Club Bot
Re: M18-04 [#permalink]
Moderator:
Math Expert
92900 posts

Powered by phpBB © phpBB Group | Emoji artwork provided by EmojiOne